Question and Answers Forum

All Questions      Topic List

Matrices and Determinants Questions

Previous in All Question      Next in All Question      

Previous in Matrices and Determinants      Next in Matrices and Determinants      

Question Number 3965 by Yozzii last updated on 25/Dec/15

Find det(A) where A is an n×n matrix  of the form   A= ((λ,1,1,…,…,…,…,1),(1,λ,1,…,…,…,…,1),(1,1,λ,…,…,…,…,1),(⋮,⋮,⋮,⋱,…,…,…,1),(⋮,⋮,⋮,⋮,⋱,…,…,1),(⋮,⋮,⋮,⋮,⋮,⋱,…,1),(⋮,⋮,⋮,⋮,⋮,⋮,⋱,1),(1,1,1,1,1,1,1,λ) )  λ=constant for leading diagonal elements  1s for all other elements.

$${Find}\:{det}\left({A}\right)\:{where}\:{A}\:{is}\:{an}\:{n}×{n}\:{matrix} \\ $$$${of}\:{the}\:{form}\: \\ $$$${A}=\begin{pmatrix}{\lambda}&{\mathrm{1}}&{\mathrm{1}}&{\ldots}&{\ldots}&{\ldots}&{\ldots}&{\mathrm{1}}\\{\mathrm{1}}&{\lambda}&{\mathrm{1}}&{\ldots}&{\ldots}&{\ldots}&{\ldots}&{\mathrm{1}}\\{\mathrm{1}}&{\mathrm{1}}&{\lambda}&{\ldots}&{\ldots}&{\ldots}&{\ldots}&{\mathrm{1}}\\{\vdots}&{\vdots}&{\vdots}&{\ddots}&{\ldots}&{\ldots}&{\ldots}&{\mathrm{1}}\\{\vdots}&{\vdots}&{\vdots}&{\vdots}&{\ddots}&{\ldots}&{\ldots}&{\mathrm{1}}\\{\vdots}&{\vdots}&{\vdots}&{\vdots}&{\vdots}&{\ddots}&{\ldots}&{\mathrm{1}}\\{\vdots}&{\vdots}&{\vdots}&{\vdots}&{\vdots}&{\vdots}&{\ddots}&{\mathrm{1}}\\{\mathrm{1}}&{\mathrm{1}}&{\mathrm{1}}&{\mathrm{1}}&{\mathrm{1}}&{\mathrm{1}}&{\mathrm{1}}&{\lambda}\end{pmatrix} \\ $$$$\lambda={constant}\:{for}\:{leading}\:{diagonal}\:{elements} \\ $$$$\mathrm{1}{s}\:{for}\:{all}\:{other}\:{elements}. \\ $$$$ \\ $$

Commented by 123456 last updated on 25/Dec/15

(λ+1)^2 −1=λ^2 +2λ+1−1=λ(λ+2)  λ+1−1=λ  (λ+2)^2 −1=λ^2 +4λ−4−1=λ^2 −4λ−5  ⋮

$$\left(\lambda+\mathrm{1}\right)^{\mathrm{2}} −\mathrm{1}=\lambda^{\mathrm{2}} +\mathrm{2}\lambda+\mathrm{1}−\mathrm{1}=\lambda\left(\lambda+\mathrm{2}\right) \\ $$$$\lambda+\mathrm{1}−\mathrm{1}=\lambda \\ $$$$\left(\lambda+\mathrm{2}\right)^{\mathrm{2}} −\mathrm{1}=\lambda^{\mathrm{2}} +\mathrm{4}\lambda−\mathrm{4}−\mathrm{1}=\lambda^{\mathrm{2}} −\mathrm{4}\lambda−\mathrm{5} \\ $$$$\vdots \\ $$

Answered by 123456 last updated on 25/Dec/15

A= determinant ((λ,1,1,…,1),(1,λ,1,…,1),(1,1,λ,…,1),(⋮,⋮,⋮,⋱,1),(1,1,1,1,λ))  A=(1/λ^(n−1) ) determinant ((λ,1,1,…,1),(λ,λ^2 ,λ,…,λ),(λ,λ,λ^2 ,…,λ),(⋮,⋮,⋮,⋱,λ),(λ,λ,λ,λ,λ^2 ))  A=(1/λ^(n−1) ) determinant ((λ,1,1,…,1),(0,(λ^2 −1),(λ−1),…,(λ−1)),(0,(λ−1),(λ^2 −1),…,(λ−1)),(⋮,⋮,⋮,⋱,(λ−1)),(0,(λ−1),(λ−1),(λ−1),(λ^2 −1)))  A=(((λ−1)/λ))^(n−1)  determinant ((λ,1,1,…,1),(0,(λ+1),1,…,1),(0,1,(λ+1),…,1),(⋮,⋮,⋮,⋱,1),(0,1,1,1,(λ+1)))  A=(((λ−1)/λ))^(n−1) (1/((λ+1)^(n−2) )) determinant ((λ,1,1,…,1),(0,(λ+1),1,…,1),(0,(λ+1),((λ+1)^2 ),…,(λ+1)),(⋮,⋮,⋮,⋱,(λ+1)),(0,(λ+1),(λ+1),(λ+1),((λ+1)^2 )))  A=(((λ−1)/λ))^(n−1) (1/((λ+1)^(n−2) )) determinant ((λ,1,1,…,1),(0,(λ+1),1,…,1),(0,0,(λ(λ+2)),…,λ),(⋮,⋮,⋮,⋱,λ),(0,0,λ,λ,(λ(λ+2))))  A=(((λ−1)/λ))^(n−1) (λ/((λ+1)^(n−2) )) determinant ((λ,1,1,…,1),(0,(λ+1),1,…,1),(0,0,(λ+2),…,1),(⋮,⋮,⋮,⋱,1),(0,0,1,1,(λ+2)))  continue

$${A}=\begin{vmatrix}{\lambda}&{\mathrm{1}}&{\mathrm{1}}&{\ldots}&{\mathrm{1}}\\{\mathrm{1}}&{\lambda}&{\mathrm{1}}&{\ldots}&{\mathrm{1}}\\{\mathrm{1}}&{\mathrm{1}}&{\lambda}&{\ldots}&{\mathrm{1}}\\{\vdots}&{\vdots}&{\vdots}&{\ddots}&{\mathrm{1}}\\{\mathrm{1}}&{\mathrm{1}}&{\mathrm{1}}&{\mathrm{1}}&{\lambda}\end{vmatrix} \\ $$$${A}=\frac{\mathrm{1}}{\lambda^{{n}−\mathrm{1}} }\begin{vmatrix}{\lambda}&{\mathrm{1}}&{\mathrm{1}}&{\ldots}&{\mathrm{1}}\\{\lambda}&{\lambda^{\mathrm{2}} }&{\lambda}&{\ldots}&{\lambda}\\{\lambda}&{\lambda}&{\lambda^{\mathrm{2}} }&{\ldots}&{\lambda}\\{\vdots}&{\vdots}&{\vdots}&{\ddots}&{\lambda}\\{\lambda}&{\lambda}&{\lambda}&{\lambda}&{\lambda^{\mathrm{2}} }\end{vmatrix} \\ $$$${A}=\frac{\mathrm{1}}{\lambda^{{n}−\mathrm{1}} }\begin{vmatrix}{\lambda}&{\mathrm{1}}&{\mathrm{1}}&{\ldots}&{\mathrm{1}}\\{\mathrm{0}}&{\lambda^{\mathrm{2}} −\mathrm{1}}&{\lambda−\mathrm{1}}&{\ldots}&{\lambda−\mathrm{1}}\\{\mathrm{0}}&{\lambda−\mathrm{1}}&{\lambda^{\mathrm{2}} −\mathrm{1}}&{\ldots}&{\lambda−\mathrm{1}}\\{\vdots}&{\vdots}&{\vdots}&{\ddots}&{\lambda−\mathrm{1}}\\{\mathrm{0}}&{\lambda−\mathrm{1}}&{\lambda−\mathrm{1}}&{\lambda−\mathrm{1}}&{\lambda^{\mathrm{2}} −\mathrm{1}}\end{vmatrix} \\ $$$${A}=\left(\frac{\lambda−\mathrm{1}}{\lambda}\right)^{{n}−\mathrm{1}} \begin{vmatrix}{\lambda}&{\mathrm{1}}&{\mathrm{1}}&{\ldots}&{\mathrm{1}}\\{\mathrm{0}}&{\lambda+\mathrm{1}}&{\mathrm{1}}&{\ldots}&{\mathrm{1}}\\{\mathrm{0}}&{\mathrm{1}}&{\lambda+\mathrm{1}}&{\ldots}&{\mathrm{1}}\\{\vdots}&{\vdots}&{\vdots}&{\ddots}&{\mathrm{1}}\\{\mathrm{0}}&{\mathrm{1}}&{\mathrm{1}}&{\mathrm{1}}&{\lambda+\mathrm{1}}\end{vmatrix} \\ $$$${A}=\left(\frac{\lambda−\mathrm{1}}{\lambda}\right)^{{n}−\mathrm{1}} \frac{\mathrm{1}}{\left(\lambda+\mathrm{1}\right)^{{n}−\mathrm{2}} }\begin{vmatrix}{\lambda}&{\mathrm{1}}&{\mathrm{1}}&{\ldots}&{\mathrm{1}}\\{\mathrm{0}}&{\lambda+\mathrm{1}}&{\mathrm{1}}&{\ldots}&{\mathrm{1}}\\{\mathrm{0}}&{\lambda+\mathrm{1}}&{\left(\lambda+\mathrm{1}\right)^{\mathrm{2}} }&{\ldots}&{\lambda+\mathrm{1}}\\{\vdots}&{\vdots}&{\vdots}&{\ddots}&{\lambda+\mathrm{1}}\\{\mathrm{0}}&{\lambda+\mathrm{1}}&{\lambda+\mathrm{1}}&{\lambda+\mathrm{1}}&{\left(\lambda+\mathrm{1}\right)^{\mathrm{2}} }\end{vmatrix} \\ $$$${A}=\left(\frac{\lambda−\mathrm{1}}{\lambda}\right)^{{n}−\mathrm{1}} \frac{\mathrm{1}}{\left(\lambda+\mathrm{1}\right)^{{n}−\mathrm{2}} }\begin{vmatrix}{\lambda}&{\mathrm{1}}&{\mathrm{1}}&{\ldots}&{\mathrm{1}}\\{\mathrm{0}}&{\lambda+\mathrm{1}}&{\mathrm{1}}&{\ldots}&{\mathrm{1}}\\{\mathrm{0}}&{\mathrm{0}}&{\lambda\left(\lambda+\mathrm{2}\right)}&{\ldots}&{\lambda}\\{\vdots}&{\vdots}&{\vdots}&{\ddots}&{\lambda}\\{\mathrm{0}}&{\mathrm{0}}&{\lambda}&{\lambda}&{\lambda\left(\lambda+\mathrm{2}\right)}\end{vmatrix} \\ $$$${A}=\left(\frac{\lambda−\mathrm{1}}{\lambda}\right)^{{n}−\mathrm{1}} \frac{\lambda}{\left(\lambda+\mathrm{1}\right)^{{n}−\mathrm{2}} }\begin{vmatrix}{\lambda}&{\mathrm{1}}&{\mathrm{1}}&{\ldots}&{\mathrm{1}}\\{\mathrm{0}}&{\lambda+\mathrm{1}}&{\mathrm{1}}&{\ldots}&{\mathrm{1}}\\{\mathrm{0}}&{\mathrm{0}}&{\lambda+\mathrm{2}}&{\ldots}&{\mathrm{1}}\\{\vdots}&{\vdots}&{\vdots}&{\ddots}&{\mathrm{1}}\\{\mathrm{0}}&{\mathrm{0}}&{\mathrm{1}}&{\mathrm{1}}&{\lambda+\mathrm{2}}\end{vmatrix} \\ $$$$\mathrm{continue} \\ $$

Commented by prakash jain last updated on 25/Dec/15

From the line in above  A=(((λ−1)/λ))^(n−1)  determinant ((λ,1,1,…,1),(0,(λ+1),1,…,1),(0,1,(λ+1),…,1),(⋮,⋮,⋮,⋱,1),(0,1,1,1,(λ+1)))  Let us say D(λ,n) is the determinant.  D(λ,n)=(((λ−1)^(n−1) )/λ^(n−2) )D(λ+1,n−1)  D(λ,2)=λ^2 −1=(λ−1)(λ+1)  D(λ,3)=(((λ−1)^2 )/λ)D(λ+1,2)=(((λ−1)^2 )/λ)[(λ+1)^2 −1]  =(λ−1)^2 (λ+2)  D(λ,4)=(((λ−1)^3 )/λ^2 )λ^2 (λ+3)=(λ−1)^3 (λ+3)  D(λ,n)=(λ−1)^(n−1) (λ+n−1)

$$\mathrm{From}\:\mathrm{the}\:\mathrm{line}\:\mathrm{in}\:\mathrm{above} \\ $$$${A}=\left(\frac{\lambda−\mathrm{1}}{\lambda}\right)^{{n}−\mathrm{1}} \begin{vmatrix}{\lambda}&{\mathrm{1}}&{\mathrm{1}}&{\ldots}&{\mathrm{1}}\\{\mathrm{0}}&{\lambda+\mathrm{1}}&{\mathrm{1}}&{\ldots}&{\mathrm{1}}\\{\mathrm{0}}&{\mathrm{1}}&{\lambda+\mathrm{1}}&{\ldots}&{\mathrm{1}}\\{\vdots}&{\vdots}&{\vdots}&{\ddots}&{\mathrm{1}}\\{\mathrm{0}}&{\mathrm{1}}&{\mathrm{1}}&{\mathrm{1}}&{\lambda+\mathrm{1}}\end{vmatrix} \\ $$$$\mathrm{Let}\:\mathrm{us}\:\mathrm{say}\:\mathrm{D}\left(\lambda,{n}\right)\:\mathrm{is}\:\mathrm{the}\:\mathrm{determinant}. \\ $$$$\mathrm{D}\left(\lambda,{n}\right)=\frac{\left(\lambda−\mathrm{1}\right)^{{n}−\mathrm{1}} }{\lambda^{{n}−\mathrm{2}} }{D}\left(\lambda+\mathrm{1},{n}−\mathrm{1}\right) \\ $$$${D}\left(\lambda,\mathrm{2}\right)=\lambda^{\mathrm{2}} −\mathrm{1}=\left(\lambda−\mathrm{1}\right)\left(\lambda+\mathrm{1}\right) \\ $$$${D}\left(\lambda,\mathrm{3}\right)=\frac{\left(\lambda−\mathrm{1}\right)^{\mathrm{2}} }{\lambda}{D}\left(\lambda+\mathrm{1},\mathrm{2}\right)=\frac{\left(\lambda−\mathrm{1}\right)^{\mathrm{2}} }{\lambda}\left[\left(\lambda+\mathrm{1}\right)^{\mathrm{2}} −\mathrm{1}\right] \\ $$$$=\left(\lambda−\mathrm{1}\right)^{\mathrm{2}} \left(\lambda+\mathrm{2}\right) \\ $$$${D}\left(\lambda,\mathrm{4}\right)=\frac{\left(\lambda−\mathrm{1}\right)^{\mathrm{3}} }{\lambda^{\mathrm{2}} }\lambda^{\mathrm{2}} \left(\lambda+\mathrm{3}\right)=\left(\lambda−\mathrm{1}\right)^{\mathrm{3}} \left(\lambda+\mathrm{3}\right) \\ $$$${D}\left(\lambda,{n}\right)=\left(\lambda−\mathrm{1}\right)^{{n}−\mathrm{1}} \left(\lambda+{n}−\mathrm{1}\right) \\ $$

Commented by Yozzii last updated on 25/Dec/15

It should be D(λ,n)=(λ−1)^(n−1) (λ+n−1) ?

$${It}\:{should}\:{be}\:{D}\left(\lambda,{n}\right)=\left(\lambda−\mathrm{1}\right)^{{n}−\mathrm{1}} \left(\lambda+{n}−\mathrm{1}\right)\:? \\ $$

Commented by prakash jain last updated on 25/Dec/15

yes. corrected.

$${yes}.\:{corrected}. \\ $$

Answered by Yozzii last updated on 26/Dec/15

Perhaps some sort of recursive   formula for ∣A_n ∣ could be found?   n=2: A_2 = ((λ,1),(1,λ) )⇒∣A_2 ∣=λ^2 −1  n=3: A_3 = ((λ,1,1),(1,λ,1),(1,1,λ) )  ⇒∣A_3 ∣=λ determinant ((λ,1),(1,λ))− determinant ((1,1),(1,λ))+ determinant ((1,λ),(1,1))  Now, − determinant ((1,1),(1,λ))= determinant ((1,λ),(1,1))  and  determinant ((λ,1),(1,λ))=∣A_2 ∣.  ∴ ∣A_3 ∣=λ∣A_2 ∣−2 determinant ((1,1),(1,λ))  ∣A_3 ∣=λ∣A_2 ∣−2(λ−1)  ∣A_3 ∣=λ(λ^2 −1)−2(λ−1)=(λ−1)(λ^2 +λ−2)  n=4:A_4 = ((λ,1,1,1),(1,λ,1,1),(1,1,λ,1),(1,1,1,λ) )  ∣A_4 ∣=λ determinant ((λ,1,1),(1,λ,1),(1,1,λ))− determinant ((1,1,1),(1,λ,1),(1,1,λ))+ determinant ((1,λ,1),(1,1,1),(1,1,λ))− determinant ((1,λ,1),(1,1,λ),(1,1,1))  Also,  determinant ((1,λ,1),(1,1,1),(1,1,λ))=− determinant ((1,1,1),(1,λ,1),(1,1,λ))=− determinant ((1,λ,1),(1,1,λ),(1,1,1))  and A_3 = determinant ((λ,1,1),(1,λ,1),(1,1,λ)).   determinant ((1,1,1),(1,λ,1),(1,1,λ))=1×(λ−1)(λ+1)−2(λ−1)  =(λ−1)(λ+1−1−1)  =(λ−1)^2   ∴∣A_4 ∣=λ∣A_3 ∣−3 determinant ((1,1,1),(1,λ,1),(1,1,λ))  ∣A_4 ∣=λ∣A_3 ∣−3(λ−1)^2     n=5: A_5 = ((λ,1,1,1,1),(1,λ,1,1,1),(1,1,λ,1,1),(1,1,1,λ,1),(1,1,1,1,λ) )  ∣A_5 ∣=λ∣A_4 ∣− determinant ((1,1,1,1),(1,λ,1,1),(1,1,λ,1),(1,1,1,λ))+ determinant ((1,λ,1,1),(1,1,1,1),(1,1,λ,1),(1,1,1,λ))− determinant ((1,λ,1,1),(1,1,λ,1),(1,1,1,1),(1,1,1,λ))+ determinant ((1,λ,1,1),(1,1,λ,1),(1,1,1,λ),(1,1,1,1))  It can be shown that ∣A_5 ∣=λ∣A_4 ∣−4 determinant ((1,1,1,1),(1,λ,1,1),(1,1,λ,1),(1,1,1,λ)).   determinant ((1,1,1,1),(1,λ,1,1),(1,1,λ,1),(1,1,1,λ))=1×∣A_3 ∣−3 determinant ((1,1,1),(1,λ,1),(1,1,λ))                            =(λ−1)[(λ^2 −2λ+1)]                            =(λ−1)^3   ∴∣A_5 ∣=λ∣A_4 ∣−4(λ−1)^3 .  I conject that ∣A_(n+1) ∣=λ∣A_n ∣−n(λ−1)^(n−1)   (n≥2)  A_2 =λ^2 −1.     This is like u_(n+1) =λu_n −n(λ−1)^(n−1)  (n≥2).    From Mr. Jain′s contribution,  ∣A_n ∣=(λ−1)^(n−1) (λ+n−1)=D(λ,n)  By my recurrence relation,  ∣A_(n+1) ∣=λ(λ−1)^(n−1) (λ+n−1)−n(λ−1)^(n−1)   ∣A_(n+1) ∣=(λ−1)^(n−1) (λ^2 +(n−1)λ−n)  ∣A_(n+1) ∣=(λ−1)^(n−1) (λ(λ−1)+n(λ−1))  ∣A_(n+1) ∣=(λ−1)^n (λ+n)=D(λ,n+1)

$${Perhaps}\:{some}\:{sort}\:{of}\:{recursive}\: \\ $$$${formula}\:{for}\:\mid{A}_{{n}} \mid\:{could}\:{be}\:{found}?\: \\ $$$${n}=\mathrm{2}:\:{A}_{\mathrm{2}} =\begin{pmatrix}{\lambda}&{\mathrm{1}}\\{\mathrm{1}}&{\lambda}\end{pmatrix}\Rightarrow\mid{A}_{\mathrm{2}} \mid=\lambda^{\mathrm{2}} −\mathrm{1} \\ $$$${n}=\mathrm{3}:\:{A}_{\mathrm{3}} =\begin{pmatrix}{\lambda}&{\mathrm{1}}&{\mathrm{1}}\\{\mathrm{1}}&{\lambda}&{\mathrm{1}}\\{\mathrm{1}}&{\mathrm{1}}&{\lambda}\end{pmatrix} \\ $$$$\Rightarrow\mid{A}_{\mathrm{3}} \mid=\lambda\begin{vmatrix}{\lambda}&{\mathrm{1}}\\{\mathrm{1}}&{\lambda}\end{vmatrix}−\begin{vmatrix}{\mathrm{1}}&{\mathrm{1}}\\{\mathrm{1}}&{\lambda}\end{vmatrix}+\begin{vmatrix}{\mathrm{1}}&{\lambda}\\{\mathrm{1}}&{\mathrm{1}}\end{vmatrix} \\ $$$${Now},\:−\begin{vmatrix}{\mathrm{1}}&{\mathrm{1}}\\{\mathrm{1}}&{\lambda}\end{vmatrix}=\begin{vmatrix}{\mathrm{1}}&{\lambda}\\{\mathrm{1}}&{\mathrm{1}}\end{vmatrix} \\ $$$${and}\:\begin{vmatrix}{\lambda}&{\mathrm{1}}\\{\mathrm{1}}&{\lambda}\end{vmatrix}=\mid{A}_{\mathrm{2}} \mid. \\ $$$$\therefore\:\mid{A}_{\mathrm{3}} \mid=\lambda\mid{A}_{\mathrm{2}} \mid−\mathrm{2}\begin{vmatrix}{\mathrm{1}}&{\mathrm{1}}\\{\mathrm{1}}&{\lambda}\end{vmatrix} \\ $$$$\mid{A}_{\mathrm{3}} \mid=\lambda\mid{A}_{\mathrm{2}} \mid−\mathrm{2}\left(\lambda−\mathrm{1}\right) \\ $$$$\mid{A}_{\mathrm{3}} \mid=\lambda\left(\lambda^{\mathrm{2}} −\mathrm{1}\right)−\mathrm{2}\left(\lambda−\mathrm{1}\right)=\left(\lambda−\mathrm{1}\right)\left(\lambda^{\mathrm{2}} +\lambda−\mathrm{2}\right) \\ $$$${n}=\mathrm{4}:{A}_{\mathrm{4}} =\begin{pmatrix}{\lambda}&{\mathrm{1}}&{\mathrm{1}}&{\mathrm{1}}\\{\mathrm{1}}&{\lambda}&{\mathrm{1}}&{\mathrm{1}}\\{\mathrm{1}}&{\mathrm{1}}&{\lambda}&{\mathrm{1}}\\{\mathrm{1}}&{\mathrm{1}}&{\mathrm{1}}&{\lambda}\end{pmatrix} \\ $$$$\mid{A}_{\mathrm{4}} \mid=\lambda\begin{vmatrix}{\lambda}&{\mathrm{1}}&{\mathrm{1}}\\{\mathrm{1}}&{\lambda}&{\mathrm{1}}\\{\mathrm{1}}&{\mathrm{1}}&{\lambda}\end{vmatrix}−\begin{vmatrix}{\mathrm{1}}&{\mathrm{1}}&{\mathrm{1}}\\{\mathrm{1}}&{\lambda}&{\mathrm{1}}\\{\mathrm{1}}&{\mathrm{1}}&{\lambda}\end{vmatrix}+\begin{vmatrix}{\mathrm{1}}&{\lambda}&{\mathrm{1}}\\{\mathrm{1}}&{\mathrm{1}}&{\mathrm{1}}\\{\mathrm{1}}&{\mathrm{1}}&{\lambda}\end{vmatrix}−\begin{vmatrix}{\mathrm{1}}&{\lambda}&{\mathrm{1}}\\{\mathrm{1}}&{\mathrm{1}}&{\lambda}\\{\mathrm{1}}&{\mathrm{1}}&{\mathrm{1}}\end{vmatrix} \\ $$$${Also},\:\begin{vmatrix}{\mathrm{1}}&{\lambda}&{\mathrm{1}}\\{\mathrm{1}}&{\mathrm{1}}&{\mathrm{1}}\\{\mathrm{1}}&{\mathrm{1}}&{\lambda}\end{vmatrix}=−\begin{vmatrix}{\mathrm{1}}&{\mathrm{1}}&{\mathrm{1}}\\{\mathrm{1}}&{\lambda}&{\mathrm{1}}\\{\mathrm{1}}&{\mathrm{1}}&{\lambda}\end{vmatrix}=−\begin{vmatrix}{\mathrm{1}}&{\lambda}&{\mathrm{1}}\\{\mathrm{1}}&{\mathrm{1}}&{\lambda}\\{\mathrm{1}}&{\mathrm{1}}&{\mathrm{1}}\end{vmatrix} \\ $$$${and}\:{A}_{\mathrm{3}} =\begin{vmatrix}{\lambda}&{\mathrm{1}}&{\mathrm{1}}\\{\mathrm{1}}&{\lambda}&{\mathrm{1}}\\{\mathrm{1}}&{\mathrm{1}}&{\lambda}\end{vmatrix}. \\ $$$$\begin{vmatrix}{\mathrm{1}}&{\mathrm{1}}&{\mathrm{1}}\\{\mathrm{1}}&{\lambda}&{\mathrm{1}}\\{\mathrm{1}}&{\mathrm{1}}&{\lambda}\end{vmatrix}=\mathrm{1}×\left(\lambda−\mathrm{1}\right)\left(\lambda+\mathrm{1}\right)−\mathrm{2}\left(\lambda−\mathrm{1}\right) \\ $$$$=\left(\lambda−\mathrm{1}\right)\left(\lambda+\mathrm{1}−\mathrm{1}−\mathrm{1}\right) \\ $$$$=\left(\lambda−\mathrm{1}\right)^{\mathrm{2}} \\ $$$$\therefore\mid{A}_{\mathrm{4}} \mid=\lambda\mid{A}_{\mathrm{3}} \mid−\mathrm{3}\begin{vmatrix}{\mathrm{1}}&{\mathrm{1}}&{\mathrm{1}}\\{\mathrm{1}}&{\lambda}&{\mathrm{1}}\\{\mathrm{1}}&{\mathrm{1}}&{\lambda}\end{vmatrix} \\ $$$$\mid{A}_{\mathrm{4}} \mid=\lambda\mid{A}_{\mathrm{3}} \mid−\mathrm{3}\left(\lambda−\mathrm{1}\right)^{\mathrm{2}} \\ $$$$ \\ $$$${n}=\mathrm{5}:\:{A}_{\mathrm{5}} =\begin{pmatrix}{\lambda}&{\mathrm{1}}&{\mathrm{1}}&{\mathrm{1}}&{\mathrm{1}}\\{\mathrm{1}}&{\lambda}&{\mathrm{1}}&{\mathrm{1}}&{\mathrm{1}}\\{\mathrm{1}}&{\mathrm{1}}&{\lambda}&{\mathrm{1}}&{\mathrm{1}}\\{\mathrm{1}}&{\mathrm{1}}&{\mathrm{1}}&{\lambda}&{\mathrm{1}}\\{\mathrm{1}}&{\mathrm{1}}&{\mathrm{1}}&{\mathrm{1}}&{\lambda}\end{pmatrix} \\ $$$$\mid{A}_{\mathrm{5}} \mid=\lambda\mid{A}_{\mathrm{4}} \mid−\begin{vmatrix}{\mathrm{1}}&{\mathrm{1}}&{\mathrm{1}}&{\mathrm{1}}\\{\mathrm{1}}&{\lambda}&{\mathrm{1}}&{\mathrm{1}}\\{\mathrm{1}}&{\mathrm{1}}&{\lambda}&{\mathrm{1}}\\{\mathrm{1}}&{\mathrm{1}}&{\mathrm{1}}&{\lambda}\end{vmatrix}+\begin{vmatrix}{\mathrm{1}}&{\lambda}&{\mathrm{1}}&{\mathrm{1}}\\{\mathrm{1}}&{\mathrm{1}}&{\mathrm{1}}&{\mathrm{1}}\\{\mathrm{1}}&{\mathrm{1}}&{\lambda}&{\mathrm{1}}\\{\mathrm{1}}&{\mathrm{1}}&{\mathrm{1}}&{\lambda}\end{vmatrix}−\begin{vmatrix}{\mathrm{1}}&{\lambda}&{\mathrm{1}}&{\mathrm{1}}\\{\mathrm{1}}&{\mathrm{1}}&{\lambda}&{\mathrm{1}}\\{\mathrm{1}}&{\mathrm{1}}&{\mathrm{1}}&{\mathrm{1}}\\{\mathrm{1}}&{\mathrm{1}}&{\mathrm{1}}&{\lambda}\end{vmatrix}+\begin{vmatrix}{\mathrm{1}}&{\lambda}&{\mathrm{1}}&{\mathrm{1}}\\{\mathrm{1}}&{\mathrm{1}}&{\lambda}&{\mathrm{1}}\\{\mathrm{1}}&{\mathrm{1}}&{\mathrm{1}}&{\lambda}\\{\mathrm{1}}&{\mathrm{1}}&{\mathrm{1}}&{\mathrm{1}}\end{vmatrix} \\ $$$${It}\:{can}\:{be}\:{shown}\:{that}\:\mid{A}_{\mathrm{5}} \mid=\lambda\mid{A}_{\mathrm{4}} \mid−\mathrm{4}\begin{vmatrix}{\mathrm{1}}&{\mathrm{1}}&{\mathrm{1}}&{\mathrm{1}}\\{\mathrm{1}}&{\lambda}&{\mathrm{1}}&{\mathrm{1}}\\{\mathrm{1}}&{\mathrm{1}}&{\lambda}&{\mathrm{1}}\\{\mathrm{1}}&{\mathrm{1}}&{\mathrm{1}}&{\lambda}\end{vmatrix}. \\ $$$$\begin{vmatrix}{\mathrm{1}}&{\mathrm{1}}&{\mathrm{1}}&{\mathrm{1}}\\{\mathrm{1}}&{\lambda}&{\mathrm{1}}&{\mathrm{1}}\\{\mathrm{1}}&{\mathrm{1}}&{\lambda}&{\mathrm{1}}\\{\mathrm{1}}&{\mathrm{1}}&{\mathrm{1}}&{\lambda}\end{vmatrix}=\mathrm{1}×\mid{A}_{\mathrm{3}} \mid−\mathrm{3}\begin{vmatrix}{\mathrm{1}}&{\mathrm{1}}&{\mathrm{1}}\\{\mathrm{1}}&{\lambda}&{\mathrm{1}}\\{\mathrm{1}}&{\mathrm{1}}&{\lambda}\end{vmatrix} \\ $$$$\:\:\:\:\:\:\:\:\:\:\:\:\:\:\:\:\:\:\:\:\:\:\:\:\:\:=\left(\lambda−\mathrm{1}\right)\left[\left(\lambda^{\mathrm{2}} −\mathrm{2}\lambda+\mathrm{1}\right)\right] \\ $$$$\:\:\:\:\:\:\:\:\:\:\:\:\:\:\:\:\:\:\:\:\:\:\:\:\:\:=\left(\lambda−\mathrm{1}\right)^{\mathrm{3}} \\ $$$$\therefore\mid{A}_{\mathrm{5}} \mid=\lambda\mid{A}_{\mathrm{4}} \mid−\mathrm{4}\left(\lambda−\mathrm{1}\right)^{\mathrm{3}} . \\ $$$${I}\:{conject}\:{that}\:\mid{A}_{{n}+\mathrm{1}} \mid=\lambda\mid{A}_{{n}} \mid−{n}\left(\lambda−\mathrm{1}\right)^{{n}−\mathrm{1}} \:\:\left({n}\geqslant\mathrm{2}\right) \\ $$$${A}_{\mathrm{2}} =\lambda^{\mathrm{2}} −\mathrm{1}.\: \\ $$$$ \\ $$$${This}\:{is}\:{like}\:{u}_{{n}+\mathrm{1}} =\lambda{u}_{{n}} −{n}\left(\lambda−\mathrm{1}\right)^{{n}−\mathrm{1}} \:\left({n}\geqslant\mathrm{2}\right). \\ $$$$ \\ $$$${From}\:{Mr}.\:{Jain}'{s}\:{contribution}, \\ $$$$\mid{A}_{{n}} \mid=\left(\lambda−\mathrm{1}\right)^{{n}−\mathrm{1}} \left(\lambda+{n}−\mathrm{1}\right)={D}\left(\lambda,{n}\right) \\ $$$${By}\:{my}\:{recurrence}\:{relation}, \\ $$$$\mid{A}_{{n}+\mathrm{1}} \mid=\lambda\left(\lambda−\mathrm{1}\right)^{{n}−\mathrm{1}} \left(\lambda+{n}−\mathrm{1}\right)−{n}\left(\lambda−\mathrm{1}\right)^{{n}−\mathrm{1}} \\ $$$$\mid{A}_{{n}+\mathrm{1}} \mid=\left(\lambda−\mathrm{1}\right)^{{n}−\mathrm{1}} \left(\lambda^{\mathrm{2}} +\left({n}−\mathrm{1}\right)\lambda−{n}\right) \\ $$$$\mid{A}_{{n}+\mathrm{1}} \mid=\left(\lambda−\mathrm{1}\right)^{{n}−\mathrm{1}} \left(\lambda\left(\lambda−\mathrm{1}\right)+{n}\left(\lambda−\mathrm{1}\right)\right) \\ $$$$\mid{A}_{{n}+\mathrm{1}} \mid=\left(\lambda−\mathrm{1}\right)^{{n}} \left(\lambda+{n}\right)={D}\left(\lambda,{n}+\mathrm{1}\right) \\ $$$$ \\ $$

Commented by Yozzii last updated on 26/Dec/15

I had hoped of solving the recursive  formula for ∣A_n ∣...Generating function?

$${I}\:{had}\:{hoped}\:{of}\:{solving}\:{the}\:{recursive} \\ $$$${formula}\:{for}\:\mid{A}_{{n}} \mid...{Generating}\:{function}? \\ $$

Terms of Service

Privacy Policy

Contact: info@tinkutara.com